LSAT and Law School Admissions Forum

Get expert LSAT preparation and law school admissions advice from PowerScore Test Preparation.

 Kristina Moen
PowerScore Staff
  • PowerScore Staff
  • Posts: 230
  • Joined: Nov 17, 2016
|
#81465
Complete Question Explanation

Weaken. The correct answer choice is (D).

This is a classic numbers and percentages question. The premise tells us about percentages, and the conclusion tells us about numbers.

The premise: "As a result of this policy, only one-quarter of all part-lime teachers now quit during their first year. However, a third of all full-time teachers now quit during their first year."

The conclusion: "More full-time than part-time teachers at Morris now quit during their first year."

This is a common argument flaw. Percentages =/= numbers. If I say that I have 50% of my discretionary income in my pocket right now, do you know how much money is in my pocket? Nope! Because you don't know my discretionary income. So you can't say that I have more money in my pocket than Joe down the street.

This is a Weaken question. We want something that weakens the conclusion. The conclusion is a comparison between full-time and part-time teachers and is talking about the current state of affairs. The premise tells us the percentages now, and the conclusion is about numbers now.

Thus, answer choices (A) and (B) will not weaken (or strengthen!) the conclusion. We don't care what teachers used to do. So you are correct that what happened "before the new policy" does nothing to attack the conclusion.

Answer choice (C), (D), and (E) are similar. So what's the difference? Let's look at some sample numbers.
Premise: Part-time teachers: 1/4 quit
Premise: Full-time teachers: 1/3 quit
Conclusion: More full-time teachers quit than part-time.

Answer choice (C): Morris High School employs more new full-time teachers than new part-time teachers. On the extreme end, what if there are 4 part-time teachers and 50 full-time teachers? Then more full-time teachers quit. This strengthens the conclusion. This is not the correct answer.

Answer choice (D): This is the correct answer choice. Morris High School employs more new part-time teachers than new full-time teachers. On the extreme end, what if there are 50 part-time teachers are 3 full-time teachers? Then more part-time teachers quit. Remember, this is a WEAKEN question. So this answer choice weakens the conclusion. It doesn't give us the numbers, but it plants the possibility that there could be more part-time teachers quitting. This is the correct answer choice.

Answer choice (E): Morris High School employs the same number of new part-time as new full-time teachers.
Okay, if there are 12 part-time and full-time teachers, then 4 full-time and 3 part-time teachers quit. Conclusion still stands. This is not the correct answer.
User avatar
 Insiya
  • Posts: 2
  • Joined: Jan 05, 2021
|
#83310
Hi,

When reviewing the answer choices for this problem, it was hard for me to see which ones were contenders and which were losers. Even after knowing that the right answer choice is (D), I am struggling to understand why that is the correct answer. Can someone please explain the process of answering this question. Thanks!
 Adam Tyson
PowerScore Staff
  • PowerScore Staff
  • Posts: 5153
  • Joined: Apr 14, 2011
|
#83335
Hi Insiya, thanks for the question! We already have a thread with a full explanation of this question, found here:

[admin note- posts moved to correct spot]

Check that out, and if the explanation doesn't do the trick for you, follow up in that thread with your question again.

Thanks, and welcome to the Forum! We're looking forward to helping you!

Get the most out of your LSAT Prep Plus subscription.

Analyze and track your performance with our Testing and Analytics Package.